0
$\begingroup$

Erdős problem 409 ("How many iterations of$n↦\phi(n)+1$ are needed before a prime is reached? Can infinitely many$n$ reach the same prime? What is the density of$n$ which reach any fixed prime?") has been extended to$\mathbb{N}^2$ inthis post, by the nonlinear recurrence$$x_n=1+\frac{\phi(x_{n-1})+\phi(x_{n-2})}2,\;\;\;x_0,x_1\gt2$$where$\phi$ is Euler's totient function.

This extension:

  • $(1)$ preserves primes as fixed points of the function;
  • $(2)$ ensures that the image of the function is contained in$\mathbb{N}$;
  • $(3)$ ensures that the function is indeed a contraction.

I ask if it is possible to do the same in$\mathbb{N}^3$.

A quite natural choice would be$$x_n=1+\frac{\phi(x_{n-1})+2\phi(x_{n-2})+3\phi(x_{n-3})}6$$but, unfortunately, it does not satisfy the second constraint everywhere.

Many thanks.

askedNov 10 at 18:03
Augusto Santi's user avatar
$\endgroup$
1
  • $\begingroup$Not sure what you are after. Of course, you can always add a floor or ceiling function to force the result to be an integer.$\endgroup$CommentedNov 10 at 18:08

0

You mustlog in to answer this question.

Start asking to get answers

Find the answer to your question by asking.

Ask question

Explore related questions

See similar questions with these tags.